Find limit using Limit's properties. 3 (x+4)2 +ex - 9 lim X-0 X

Answers

Answer 1

The limit of the function (x+4)^2 + e^x - 9 as x approaches 0 is equal to 8.

To find the limit of a function as x approaches a specific value, we can use various limit properties. In this case, we are trying to find the limit of the function (x+4)^2 + e^x - 9 as x approaches 0.

Using limit properties, we can break down the function and evaluate each term separately.

The first term, (x+4)^2, represents a polynomial function. When x approaches 0, the term simplifies to (0+4)^2 = 4^2 = 16.

The second term, e^x, represents the exponential function. As x approaches 0, e^x approaches 1, since e^0 = 1.

The third term, -9, is a constant term and does not depend on x. Thus, the limit of -9 as x approaches 0 is -9.

By applying the limit properties, we can combine these individual limits to find the overall limit of the function. In this case, the limit of the given function as x approaches 0 is the sum of the limits of each term: 16 + 1 - 9 = 8.

Learn more about polynomial here:

https://brainly.com/question/11536910

#SPJ11


Related Questions

Finance. Suppose that $3,900 is invested at 4.2% annual interest rate, compounded monthly. How much money will be in the account in (A) 11 months? (B) 14 years

Answers

a. the amount in the account after 11 months is $4,056.45.

b. the amount in the account after 14 years is $7,089.88.

Given data:

Principal amount (P) = $3,900

Annual interest rate (r) = 4.2% per annum

Number of times the interest is compounded in a year (n) = 12 (since the interest is compounded monthly)

Let's first solve for (A)

How much money will be in the account in 11 months?

Time period (t) = 11/12 year (since the interest is compounded monthly)

We need to calculate the amount (A) after 11 months.

To find:

Amount (A) after 11 months using the formula A = [tex]P(1 + r/n)^{(n*t)}[/tex]

where P = Principal amount, r = annual interest rate, n = number of times the interest is compounded in a year, and t = time period.

A = [tex]3900(1 + 0.042/12)^{(12*(11/12))}[/tex]

A = [tex]3900(1.0035)^{11}[/tex]

A = $4,056.45

Next, let's solve for (B)

How much money will be in the account in 14 years?

Time period (t) = 14 years

We need to calculate the amount (A) after 14 years.

To find:

Amount (A) after 14 years using the formula A = [tex]P(1 + r/n)^{(n*t)}[/tex]

where P = Principal amount, r = annual interest rate, n = number of times the interest is compounded in a year, and t = time period.

A = [tex]3900(1 + 0.042/12)^{(12*14)}[/tex]

A =[tex]3900(1.0035)^{168}[/tex]

A = $7,089.88

To learn more about Principal amount, refer:-

https://brainly.com/question/11566183

#SPJ11

Differentiate the following function. y = O (x-3)* > O (x-3)e* +8 O(x-3)x4 ex None of the above answers D Question 2 Differentiate the following function. y = x³ex O y'= (x³ + 3x²)e* Oy' = (x³ + 3x²)e²x O y'= (2x³ + 3x²)ex None of the above answers. Question 3 Differentiate the following function. y = √√x³ + 4 O 3x² 2(x + 4)¹/3 o'y' = 2x³ 2(x+4)¹/2 3x² 2(x³ + 4)¹/2 O None of the above answers Question 4 Find the derivative of the following function." y = 24x O y' = 24x+2 In2 Oy² = 4x+² In 2 Oy' = 24x+2 en 2 None of the above answers.

Answers

The first three questions involve differentiating given functions.  Question 1 - None of the above answers; Question 2 - y' = (x³ + 3x²)e*; Question 3 - None of the above answers. Question 4 asks for the derivative of y = 24x, and the correct answer is y' = 24.

Question 1: The given function is y = O (x-3)* > O (x-3)e* +8 O(x-3)x4 ex. The notation used is unclear, so it is difficult to determine the correct differentiation. However, none of the provided options seem to match the given function, so the answer is "None of the above answers."

Question 2: The given function is y = x³ex. To find its derivative, we apply the product rule and the chain rule. Using the product rule, we differentiate the terms separately and combine them. The derivative of x³ is 3x², and the derivative of ex is ex. Thus, the derivative of the given function is y' = (x³ + 3x²)e*.

Question 3: The given function is y = √√x³ + 4. To differentiate this function, we apply the chain rule. The derivative of √√x³ + 4 can be found by differentiating the inner function, which is x³ + 4. The derivative of x³ + 4 is 3x², and applying the chain rule, the derivative of √√x³ + 4 becomes 3x² * 2(x + 4)¹/2. Thus, the correct answer is "3x² * 2(x + 4)¹/2."

Question 4: The given function is y = 24x. To find its derivative, we differentiate it with respect to x. The derivative of 24x is simply 24, as the derivative of a constant multiplied by x is the constant. Therefore, the correct answer is y' = 24.

Learn more about derivative here: https://brainly.com/question/32963989

#SPJ11

A pair of shoes has been discounted by 12%. If the sale price is $120, what was the original price of the shoes? [2] (b) The mass of the proton is 1.6726 x 10-27 kg and the mass of the electron is 9.1095 x 10-31 kg. Calculate the ratio of the mass of the proton to the mass of the electron. Write your answer in scientific notation correct to 3 significant figures. [2] (c) Gavin has 50-cent, one-dollar and two-dollar coins in the ratio of 8:1:2, respectively. If 30 of Gavin's coins are two-dollar, how many 50-cent and one-dollar coins does Gavin have? [2] (d) A model city has a scale ratio of 1: 1000. Find the actual height in meters of a building that has a scaled height of 8 cm. [2] (e) A house rent is divided among Akhil, Bob and Carlos in the ratio of 3:7:6. If Akhil's [2] share is $150, calculate the other shares.

Answers

The correct answer is Bob's share is approximately $350 and Carlos's share is approximately $300.

(a) To find the original price of the shoes, we can use the fact that the sale price is 88% of the original price (100% - 12% discount).

Let's denote the original price as x.

The equation can be set up as:

0.88x = $120

To find x, we divide both sides of the equation by 0.88:

x = $120 / 0.88

Using a calculator, we find:

x ≈ $136.36

Therefore, the original price of the shoes was approximately $136.36.

(b) To calculate the ratio of the mass of the proton to the mass of theelectron, we divide the mass of the proton by the mass of the electron.

Mass of proton: 1.6726 x 10^(-27) kg

Mass of electron: 9.1095 x 10^(-31) kg

Ratio = Mass of proton / Mass of electron

Ratio = (1.6726 x 10^(-27)) / (9.1095 x 10^(-31))

Performing the division, we get:

Ratio ≈ 1837.58

Therefore, the ratio of the mass of the proton to the mass of the electron is approximately 1837.58.

(c) Let's assume the common ratio of the coins is x. Then, we can set up the equation:

8x + x + 2x = 30

Combining like terms:11x = 30

Dividing both sides by 11:x = 30 / 11

Since the ratio of 50-cent, one-dollar, and two-dollar coins is 8:1:2, we can multiply the value of x by the respective ratios to find the number of each coin:

50-cent coins: 8x = 8 * (30 / 11)

one-dollar coins: 1x = 1 * (30 / 11)

Calculating the values:

50-cent coins ≈ 21.82

one-dollar coins ≈ 2.73

Since we cannot have fractional coins, we round the values:

50-cent coins ≈ 22

one-dollar coins ≈ 3

Therefore, Gavin has approximately 22 fifty-cent coins and 3 one-dollar coins.

(d) The scale ratio of the model city is 1:1000. This means that 1 cm on the model represents 1000 cm (or 10 meters) in actuality.

Given that the scaled height of the building is 8 cm, we can multiply it by the scale ratio to find the actual height:

Actual height = Scaled height * Scale ratio

Actual height = 8 cm * 10 meters/cm

Calculating the value:

Actual height = 80 meters

Therefore, the actual height of the building is 80 meters.

(e) The ratio of Akhil's share to the total share is 3:16 (3 + 7 + 6 = 16).

Since Akhil's share is $150, we can calculate the total share using the ratio:

Total share = (Total amount / Akhil's share) * Akhil's share

Total share = (16 / 3) * $150

Calculating the value:

Total share ≈ $800

To find Bob's share, we can calculate it using the ratio:

Bob's share = (Bob's ratio / Total ratio) * Total share

Bob's share = (7 / 16) * $800

Calculating the value:

Bob's share ≈ $350

To find Carlos's share, we can calculate it using the ratio:

Carlos's share = (Carlos's ratio / Total ratio) * Total share

Carlos's share = (6 / 16) * $800

Calculating the value:

Carlos's share ≈ $300

Therefore, Bob's share is approximately $350 and Carlos's share is approximately $300.

Learn more about profit and loss here:

https://brainly.com/question/26483369

#SPJ11

You will begin with a relatively standard calculation Consider a concave spherical mirror with a radius of curvature equal to 60.0 centimeters. An object 6 00 centimeters tall is placed along the axis of the mirror, 45.0 centimeters from the mirror. You are to find the location and height of the image. Part G What is the magnification n?. Part J What is the value of s' obtained from this new equation? Express your answer in terms of s.

Answers

The magnification n can be found by using the formula n = -s'/s, where s' is the image distance and s is the object distance. The value of s' obtained from this new equation can be found by rearranging the formula to s' = -ns.


To find the magnification n, we can use the formula n = -s'/s, where s' is the image distance and s is the object distance. In this case, the object is placed 45.0 centimeters from the mirror, so s = 45.0 cm. The magnification can be found by calculating the ratio of the image distance to the object distance. By rearranging the formula, we get n = -s'/s.

To find the value of s' obtained from this new equation, we can rearrange the formula n = -s'/s to solve for s'. This gives us s' = -ns. By substituting the value of n calculated earlier, we can find the value of s'. The negative sign indicates that the image is inverted.

Using the given values, we can now calculate the magnification and the value of s'. Plugging in s = 45.0 cm, we find that s' = -ns = -(2/3)(45.0 cm) = -30.0 cm. This means that the image is located 30.0 centimeters from the mirror and is inverted compared to the object.

To know more about Image visit.

https://brainly.com/question/30725545

#SPJ11

Select the correct answer.
Which of the following represents a factor from the expression given?
5(3x² +9x) -14
O 15x²
O5
O45x
O 70

Answers

The factor from the expression 5(3x² + 9x) - 14 is not listed among the options you provided. However, I can help you simplify the expression and identify the factors within it.

To simplify the expression, we can distribute the 5 to both terms inside the parentheses:

5(3x² + 9x) - 14 = 15x² + 45x - 14

From this simplified expression, we can identify the factors as follows:

15x²: This represents the term with the variable x squared.

45x: This represents the term with the variable x.

-14: This represents the constant term.

Therefore, the factors from the expression are 15x², 45x, and -14.

Find a general solution to the differential equation. 1 31 +4y=2 tan 4t 2 2 The general solution is y(t) = C₁ cos (41) + C₂ sin (41) - 25 31 e -IN Question 4, 4.6.17 GEXCES 1 In sec (4t)+ tan (41) cos (41) 2 < Jona HW Sc Poi Find a general solution to the differential equation. 1 3t y"+2y=2 tan 2t- e 2 3t The general solution is y(t) = C₁ cos 2t + C₂ sin 2t - e 26 1 In |sec 2t + tan 2t| cos 2t. --

Answers

The general solution to the given differential equation is y(t) = [tex]C_{1}\ cos{2t}\ + C_{2} \ sin{2t} - e^{2/3t}[/tex], where C₁ and C₂ are constants.

The given differential equation is a second-order linear homogeneous equation with constant coefficients. Its characteristic equation is [tex]r^2[/tex] + 2 = 0, which has complex roots r = ±i√2. Since the roots are complex, the general solution will involve trigonometric functions.

Let's assume the solution has the form y(t) = [tex]e^{rt}[/tex]. Substituting this into the differential equation, we get [tex]r^2e^{rt} + 2e^{rt} = 0[/tex]. Dividing both sides by [tex]e^{rt}[/tex], we obtain the characteristic equation [tex]r^2[/tex] + 2 = 0.

The complex roots of the characteristic equation are r = ±i√2. Using Euler's formula, we can rewrite these roots as r₁ = i√2 and r₂ = -i√2. The general solution for the homogeneous equation is y_h(t) = [tex]C_{1}e^{r_{1} t} + C_{2}e^{r_{2}t}[/tex]

Next, we need to find the particular solution for the given non-homogeneous equation. The non-homogeneous term includes a tangent function and an exponential term. We can use the method of undetermined coefficients to find a particular solution. Assuming y_p(t) has the form [tex]A \tan{2t} + Be^{2/3t}[/tex], we substitute it into the differential equation and solve for the coefficients A and B.

After finding the particular solution, we can add it to the general solution of the homogeneous equation to obtain the general solution of the non-homogeneous equation: y(t) = y_h(t) + y_p(t). Simplifying the expression, we arrive at the general solution y(t) = C₁ cos(2t) + C₂ sin(2t) - [tex]e^{2/3t}[/tex], where C₁ and C₂ are arbitrary constants determined by initial conditions or boundary conditions.

Learn more about differential equation here:

https://brainly.com/question/32538700

#SPJ11

Consider the function f(x) = 2x³ + 30x² 54x + 5. For this function there are three important open intervals: (− [infinity], A), (A, B), and (B, [infinity]) where A and B are the critical numbers. Find A and B For each of the following open intervals, tell whether f(x) is increasing or decreasing. ( − [infinity], A): Decreasing (A, B): Increasing (B, [infinity]): Decreasing

Answers

The critical numbers for the given function f(x) = 2x³ + 30x² + 54x + 5 are A = -1 and B = -9. Also, it is obtained that (-∞, A): Decreasing, (A, B): Decreasing, (B, ∞): Increasing.

To find the critical numbers A and B for the function f(x) = 2x³ + 30x² + 54x + 5, we need to find the values of x where the derivative of the function equals zero or is undefined. Let's go through the steps:

Find the derivative of f(x):
f'(x) = 6x² + 60x + 54
Set the derivative equal to zero and solve for x:
6x² + 60x + 54 = 0
Divide the equation by 6 to simplify:
x² + 10x + 9 = 0
Factor the quadratic equation:
(x + 1)(x + 9) = 0
Setting each factor equal to zero:
x + 1 = 0 -> x = -1
x + 9 = 0 -> x = -9

So the critical numbers are A = -1 and B = -9.

Now let's determine whether the function is increasing or decreasing in each of the open intervals:

(-∞, A) = (-∞, -1):

To determine if the function is increasing or decreasing, we can analyze the sign of the derivative.

Substitute a value less than -1, say x = -2, into the derivative:

f'(-2) = 6(-2)² + 60(-2) + 54 = 24 - 120 + 54 = -42

Since the derivative is negative, f(x) is decreasing in the interval (-∞, -1).

(A, B) = (-1, -9):

Similarly, substitute a value between -1 and -9, say x = -5, into the derivative:

f'(-5) = 6(-5)² + 60(-5) + 54 = 150 - 300 + 54 = -96

The derivative is negative, indicating that f(x) is decreasing in the interval (-1, -9).

(B, ∞) = (-9, ∞):

Substitute a value greater than -9, say x = 0, into the derivative:

f'(0) = 6(0)² + 60(0) + 54 = 54

The derivative is positive, implying that f(x) is increasing in the interval (-9, ∞).

To summarize:

A = -1

B = -9

(-∞, A): Decreasing

(A, B): Decreasing

(B, ∞): Increasing

To learn more about derivative visit:

brainly.com/question/32963989

#SPJ11

Installment Loan
How much of the first
$5000.00
payment for the
installment loan
5 years
12% shown in the table will
go towards interest?
Principal
Term Length
Interest Rate
Monthly Payment $111.00
A. $50.00
C. $65.00
B. $40.00
D. $61.00

Answers

The amount out of the first $ 111 payment that will go towards interest would be A. $ 50. 00.

How to find the interest portion ?

For an installment loan, the first payment is mostly used to pay off the interest. The interest portion of the loan payment can be calculated using the formula:

Interest = Principal x Interest rate / Number of payments per year

Given the information:

Principal is $5000

the Interest rate is 12% per year

number of payments per year is 12

The interest is therefore :

= 5, 000 x 0. 12 / 12 months

= $ 50

Find out more on interest at https://brainly.com/question/31393654

#SPJ1

Write out the form of the partial fraction expansion of the function. Do not determine the numerical values of the coefficients. 7x (a) (x + 2)(3x + 4) X 10 (b) x3 + 10x² + 25x Need Help? Watch It

Answers

Partial fraction expansion as:

(x³+ 10x²+ 25x) = A / x + B / (x + 5) + C / (x + 5)²

Again, A, B, and C are constants that we need to determine.

Let's break down the partial fraction expansions for the given functions:

(a) 7x / [(x + 2)(3x + 4)]

To find the partial fraction expansion of this expression, we need to factor the denominator first:

(x + 2)(3x + 4)

Next, we express the expression as a sum of partial fractions:

7x / [(x + 2)(3x + 4)] = A / (x + 2) + B / (3x + 4)

Here, A and B are constants that we need to determine.

(b) (x³ + 10x² + 25x)

Since this expression is a polynomial, we don't need to factor anything. We can directly write its partial fraction expansion as:

(x³+ 10x²+ 25x) = A / x + B / (x + 5) + C / (x + 5)²

Again, A, B, and C are constants that we need to determine.

Remember that the coefficients A, B, and C are specific values that need to be determined by solving a system of equations.

Learn more about partial fraction expansions here:

https://brainly.com/question/31707489

#SPJ11

Use limits to find the derivative function f' for the function f. b. Evaluate f'(a) for the given values of a. 2 f(x) = 4 2x+1;a= a. f'(x) = I - 3'

Answers

the derivative function of f(x) is f'(x) = 8.To find f'(a) when a = 2, simply substitute 2 for x in the derivative function:

f'(2) = 8So the value of f'(a) for a = 2 is f'(2) = 8.

The question is asking for the derivative function, f'(x), of the function f(x) = 4(2x + 1) using limits, as well as the value of f'(a) when a = 2.

To find the derivative function, f'(x), using limits, follow these steps:

Step 1:

Write out the formula for the derivative of f(x):f'(x) = lim h → 0 [f(x + h) - f(x)] / h

Step 2:

Substitute the function f(x) into the formula:

f'(x) = lim h → 0 [f(x + h) - f(x)] / h = lim h → 0 [4(2(x + h) + 1) - 4(2x + 1)] / h

Step 3:

Simplify the expression inside the limit:

f'(x) = lim h → 0 [8x + 8h + 4 - 8x - 4] / h = lim h → 0 (8h / h) + (0 / h) = 8

Step 4:

Write the final answer: f'(x) = 8

Therefore, the derivative function of f(x) is f'(x) = 8.To find f'(a) when a = 2, simply substitute 2 for x in the derivative function:

f'(2) = 8So the value of f'(a) for a = 2 is f'(2) = 8.

learn more about derivative function here

https://brainly.com/question/12047216

#SPJ11

The answer above is NOT correct. Find the orthogonal projection of onto the subspace W of R4 spanned by -1632 -2004 projw(v) = 10284 -36 v = -1 -16] -4 12 16 and 4 5 -26

Answers

Therefore, the orthogonal projection of v onto the subspace W is approximately (-32.27, -64.57, -103.89, -16.71).

To find the orthogonal projection of vector v onto the subspace W spanned by the given vectors, we can use the formula:

projₓy = (y⋅x / ||x||²) * x

where x represents the vectors spanning the subspace, y represents the vector we want to project, and ⋅ denotes the dot product.

Let's calculate the orthogonal projection:

Step 1: Normalize the spanning vectors.

First, we normalize the spanning vectors of W:

u₁ = (-1/√6, -2/√6, -3/√6, -2/√6)

u₂ = (4/√53, 5/√53, -26/√53)

Step 2: Calculate the dot product.

Next, we calculate the dot product of the vector we want to project, v, with the normalized spanning vectors:

v⋅u₁ = (-1)(-1/√6) + (-16)(-2/√6) + (-4)(-3/√6) + (12)(-2/√6)

= 1/√6 + 32/√6 + 12/√6 - 24/√6

= 21/√6

v⋅u₂ = (-1)(4/√53) + (-16)(5/√53) + (-4)(-26/√53) + (12)(0/√53)

= -4/√53 - 80/√53 + 104/√53 + 0

= 20/√53

Step 3: Calculate the projection.

Finally, we calculate the orthogonal projection of v onto the subspace W:

projW(v) = (v⋅u₁) * u₁ + (v⋅u₂) * u₂

= (21/√6) * (-1/√6, -2/√6, -3/√6, -2/√6) + (20/√53) * (4/√53, 5/√53, -26/√53)

= (-21/6, -42/6, -63/6, -42/6) + (80/53, 100/53, -520/53)

= (-21/6 + 80/53, -42/6 + 100/53, -63/6 - 520/53, -42/6)

= (-10284/318, -20544/318, -33036/318, -5304/318)

≈ (-32.27, -64.57, -103.89, -16.71)

To know more about orthogonal projection,

https://brainly.com/question/30031077

#SPJ11

In the diagram below, how many different paths from A to B are possible if you can only move forward and down? A 4 B 3. A band consisting of 3 musicians must include at least 2 guitar players. If 7 pianists and 5 guitar players are trying out for the band, then the maximum number of ways that the band can be selected is 50₂ +503 C₂ 7C1+5C3 C₂ 7C15C17C2+7C3 D5C₂+50₁ +5Co

Answers

There are 35 different paths from A to B in the diagram. This can be calculated using the multinomial rule, which states that the number of possible arrangements of n objects, where there are r1 objects of type A, r2 objects of type B, and so on, is given by:

n! / r1! * r2! * ...

In this case, we have n = 7 objects (the 4 horizontal moves and the 3 vertical moves), r1 = 4 objects of type A (the horizontal moves), and r2 = 3 objects of type B (the vertical moves). So, the number of paths is:

7! / 4! * 3! = 35

The multinomial rule can be used to calculate the number of possible arrangements of any number of objects. In this case, we have 7 objects, which we can arrange in 7! ways. However, some of these arrangements are the same, since we can move the objects around without changing the path. For example, the path AABB is the same as the path BABA. So, we need to divide 7! by the number of ways that we can arrange the objects without changing the path.

The number of ways that we can arrange 4 objects of type A and 3 objects of type B is 7! / 4! * 3!. This gives us 35 possible paths from A to B.

To learn more about multinomial rule click here : brainly.com/question/32616196

#SPJ11

If y(x) is the solution to the initial value problem y' - y = x² + x, y(1) = 2. then the value y(2) is equal to: 06 02 0-1

Answers

To find the value of y(2), we need to solve the initial value problem and evaluate the solution at x = 2.

The given initial value problem is:

y' - y = x² + x

y(1) = 2

First, let's find the integrating factor for the homogeneous equation y' - y = 0. The integrating factor is given by e^(∫-1 dx), which simplifies to [tex]e^(-x).[/tex]

Next, we multiply the entire equation by the integrating factor: [tex]e^(-x) * y' - e^(-x) * y = e^(-x) * (x² + x)[/tex]

Applying the product rule to the left side, we get:

[tex](e^(-x) * y)' = e^(-x) * (x² + x)[/tex]

Integrating both sides with respect to x, we have:

∫ ([tex]e^(-x)[/tex]* y)' dx = ∫[tex]e^(-x)[/tex] * (x² + x) dx

Integrating the left side gives us:

[tex]e^(-x)[/tex] * y = -[tex]e^(-x)[/tex]* (x³/3 + x²/2) + C1

Simplifying the right side and dividing through by e^(-x), we get:

y = -x³/3 - x²/2 +[tex]Ce^x[/tex]

Now, let's use the initial condition y(1) = 2 to solve for the constant C:

2 = -1/3 - 1/2 + [tex]Ce^1[/tex]

2 = -5/6 + Ce

C = 17/6

Finally, we substitute the value of C back into the equation and evaluate y(2):

y = -x³/3 - x²/2 + (17/6)[tex]e^x[/tex]

y(2) = -(2)³/3 - (2)²/2 + (17/6)[tex]e^2[/tex]

y(2) = -8/3 - 2 + (17/6)[tex]e^2[/tex]

y(2) = -14/3 + (17/6)[tex]e^2[/tex]

So, the value of y(2) is -14/3 + (17/6)[tex]e^2.[/tex]

Learn more about integrals here:

https://brainly.com/question/30094386

#SPJ11

Summer Rental Lynn and Judy are pooling their savings to rent a cottage in Maine for a week this summer. The rental cost is $950. Lynn’s family is joining them, so she is paying a larger part of the cost. Her share of the cost is $250 less than twice Judy’s. How much of the rental fee is each of them paying?

Answers

Lynn is paying $550 and Judy is paying $400 for the cottage rental in Maine this summer.

To find out how much of the rental fee Lynn and Judy are paying, we have to create an equation that shows the relationship between the variables in the problem.

Let L be Lynn's share of the cost, and J be Judy's share of the cost.

Then we can translate the given information into the following system of equations:

L + J = 950 (since they are pooling their savings to pay the $950 rental cost)

L = 2J - 250 (since Lynn is paying $250 less than twice Judy's share)

To solve this system, we can use substitution.

We'll solve the second equation for J and then substitute that expression into the first equation:

L = 2J - 250

L + 250 = 2J

L/2 + 125 = J

Now we can substitute that expression for J into the first equation and solve for L:

L + J = 950

L + L/2 + 125 = 950

3L/2 = 825L = 550

So, Lynn is paying $550 and Judy is paying $400.

Learn more about substitution visit:

brainly.com/question/1132161

#SPJ11

Someone help please!

Answers

The graph A is the graph of the function [tex]f(x) = -x^4 + 9[/tex].

What is the end behavior of a function?

The end behavior of a function refers to how the function behaves as the input variable approaches positive or negative infinity.

The function in this problem is given as follows:

[tex]f(x) = -x^4 + 9[/tex]

It has a negative leading coefficient with an even root, meaning that the function will approach negative infinity both to the left and to the right of the graph.

Hence the graph A is the graph of the function [tex]f(x) = -x^4 + 9[/tex].

More can be learned about the end behavior of a function at brainly.com/question/1365136

#SPJ1

b) V = (y² – x, z² + y, x − 3z) Compute F(V) S(0,3)

Answers

To compute F(V) at the point S(0,3), where V = (y² – x, z² + y, x − 3z), we substitute the values x = 0, y = 3, and z = 0 into the components of V. This yields the vector F(V) at the given point.

Given V = (y² – x, z² + y, x − 3z) and the point S(0,3), we need to compute F(V) at that point.

Substituting x = 0, y = 3, and z = 0 into the components of V, we have:

V = ((3)² - 0, (0)² + 3, 0 - 3(0))

  = (9, 3, 0)

This means that the vector V evaluates to (9, 3, 0) at the point S(0,3).

Now, to compute F(V), we need to apply the transformation F to the vector V. The specific definition of F is not provided in the question. Therefore, without further information about the transformation F, we cannot determine the exact computation of F(V) at the point S(0,3).

In summary, at the point S(0,3), the vector V evaluates to (9, 3, 0). However, the computation of F(V) cannot be determined without the explicit definition of the transformation F.

Learn more about vector here:

https://brainly.com/question/24256726

#SPJ11

In solving the beam equation, you determined that the general solution is 1 y v=ối 791-x-³ +x. Given that y''(1) = 3 determine 9₁

Answers

Given that y''(1) = 3, determine the value of 9₁.

In order to solve for 9₁ given that y''(1) = 3,

we need to start by differentiating y(x) twice with respect to x.

y(x) = c₁(x-1)³ + c₂(x-1)

where c₁ and c₂ are constantsTaking the first derivative of y(x), we get:

y'(x) = 3c₁(x-1)² + c₂

Taking the second derivative of y(x), we get:

y''(x) = 6c₁(x-1)

Let's substitute x = 1 in the expression for y''(x):

y''(1) = 6c₁(1-1)y''(1)

= 0

However, we're given that y''(1) = 3.

This is a contradiction.

Therefore, there is no value of 9₁ that satisfies the given conditions.

To know more about  derivative visit:

https://brainly.com/question/25324584

#SPJ11

If G is a complementry graph, with n vertices Prove that it is either n=0 mod 4 or either n = 1 modu

Answers

If G is a complementary graph with n vertices, then n must satisfy either n ≡ 0 (mod 4) or n ≡ 1 (mod 4).

To prove this statement, we consider the definition of a complementary graph. In a complementary graph, every edge that is not in the original graph is present in the complementary graph, and every edge in the original graph is not present in the complementary graph.

Let G be a complementary graph with n vertices. The original graph has C(n, 2) = n(n-1)/2 edges, where C(n, 2) represents the number of ways to choose 2 vertices from n. The complementary graph has C(n, 2) - E edges, where E is the number of edges in the original graph.

Since G is complementary, the total number of edges in both G and its complement is equal to the number of edges in the complete graph with n vertices, which is C(n, 2) = n(n-1)/2.

We can now express the number of edges in the complementary graph as: E = n(n-1)/2 - E.

Simplifying the equation, we get 2E = n(n-1)/2.

This equation can be rearranged as n² - n - 4E = 0.

Applying the quadratic formula to solve for n, we get n = (1 ± √(1+16E))/2.

Since n represents the number of vertices, it must be a non-negative integer. Therefore, n = (1 ± √(1+16E))/2 must be an integer.

Analyzing the two possible cases:

If n is even (n ≡ 0 (mod 2)), then n = (1 + √(1+16E))/2 is an integer if and only if √(1+16E) is an odd integer. This occurs when 1+16E is a perfect square of an odd integer.

If n is odd (n ≡ 1 (mod 2)), then n = (1 - √(1+16E))/2 is an integer if and only if √(1+16E) is an even integer. This occurs when 1+16E is a perfect square of an even integer.

In both cases, the values of n satisfy the required congruence conditions: either n ≡ 0 (mod 4) or n ≡ 1 (mod 4).

Learn more about quadratic formula here:

https://brainly.com/question/22364785

#SPJ11

Use Laplace transform to solve the following system: a' (t) = -3x(t)- 2y(t) + 2 y' (t) = 2x(t) + y(t) r(0) = 1, y(0) = 0.

Answers

To solve the given system of differential equations using Laplace transform, we will transform the differential equations into algebraic equations and then solve for the Laplace transforms of the variables.

Let's denote the Laplace transforms of a(t) and y(t) as A(s) and Y(s), respectively.

Applying the Laplace transform to the given system, we obtain:

sA(s) - a(0) = -3X(s) - 2Y(s)

sY(s) - y(0) = 2X(s) + Y(s)

Using the initial conditions, we have a(0) = 1 and y(0) = 0. Substituting these values into the equations, we get:

sA(s) - 1 = -3X(s) - 2Y(s)

sY(s) = 2X(s) + Y(s)

Rearranging the equations, we have:

sA(s) + 3X(s) + 2Y(s) = 1

sY(s) - Y(s) = 2X(s)

Solving for X(s) and Y(s) in terms of A(s), we get:

X(s) = (1/(2s+3)) * (sA(s) - 1)

Y(s) = (1/(s-1)) * (2X(s))

Substituting the expression for X(s) into Y(s), we have:

Y(s) = (1/(s-1)) * (2/(2s+3)) * (sA(s) - 1)

Now, we can take the inverse Laplace transform to find the solutions for a(t) and y(t).

To know more about Laplace transform click here: brainly.com/question/30759963

#SPJ11

This table represents a quadratic function with a vertex at (1, 0). What is the
average rate of change for the interval from x= 5 to x = 6?
A 9
OB. 5
C. 7
D. 25
X
-
2
3
4
5
0
4
9
16
P

Answers

Answer: 9

Step-by-step explanation:

Answer:To find the average rate of change for the interval from x = 5 to x = 6, we need to calculate the change in the function values over that interval and divide it by the change in x.

Given the points (5, 0) and (6, 4), we can calculate the change in the function values:

Change in y = 4 - 0 = 4

Change in x = 6 - 5 = 1

Average rate of change = Change in y / Change in x = 4 / 1 = 4

Therefore, the correct answer is 4. None of the given options (A, B, C, or D) match the correct answer.

Step-by-step explanation:

Classroom Assignment Name Date Solve the problem. 1) 1) A projectile is thrown upward so that its distance above the ground after t seconds is h=-1212 + 360t. After how many seconds does it reach its maximum height? 2) The number of mosquitoes M(x), in millions, in a certain area depends on the June rainfall 2) x, in inches: M(x) = 4x-x2. What rainfall produces the maximum number of mosquitoes? 3) The cost in millions of dollars for a company to manufacture x thousand automobiles is 3) given by the function C(x)=3x2-24x + 144. Find the number of automobiles that must be produced to minimize the cost. 4) The profit that the vendor makes per day by selling x pretzels is given by the function P(x) = -0.004x² +2.4x - 350. Find the number of pretzels that must be sold to maximize profit.

Answers

The projectile reaches its height after 30 seconds, 2 inches of rainfall produces number of mosquitoes, 4 thousand automobiles needed to minimize cost, and 300 pretzels must be sold to maximize profit.

To find the time it takes for the projectile to reach its maximum height, we need to determine the time at which the velocity becomes zero. Since the projectile is thrown upward, the initial velocity is positive and the acceleration is negative due to gravity. The velocity function is v(t) = h'(t) = 360 - 12t. Setting v(t) = 0 and solving for t, we get 360 - 12t = 0. Solving this equation, we find t = 30 seconds. Therefore, the projectile reaches its maximum height after 30 seconds.To find the rainfall that produces the maximum number of mosquitoes, we need to maximize the function M(x) = 4x - x^2. Since this is a quadratic function, we can find the maximum by determining the vertex. The x-coordinate of the vertex can be found using the formula x = -b/(2a), where a = -1 and b = 4. Plugging these values into the formula, we get x = -4/(2*(-1)) = 2 inches of rainfall. Therefore, 2 inches of rainfall produces the maximum number of mosquitoes.

To minimize the cost of manufacturing automobiles, we need to find the number of automobiles that minimizes the cost function C(x) = 3x^2 - 24x + 144. Since this is a quadratic function, the minimum occurs at the vertex. The x-coordinate of the vertex can be found using the formula x = -b/(2a), where a = 3 and b = -24. Plugging these values into the formula, we get x = -(-24)/(2*3) = 4 thousand automobiles. Therefore, 4 thousand automobiles must be produced to minimize the cost.

To maximize the profit from selling pretzels, we need to find the number of pretzels that maximizes the profit function P(x) = -0.004x^2 + 2.4x - 350. Since this is a quadratic function, the maximum occurs at the vertex. The x-coordinate of the vertex can be found using the formula x = -b/(2a), where a = -0.004 and b = 2.4. Plugging these values into the formula, we get x = -2.4/(2*(-0.004)) = 300 pretzels. Therefore, 300 pretzels must be sold to maximize the profit.

To learn more about projectile click here : brainly.com/question/28043302

#SPJ11

ind the differential dy. y=ex/2 dy = (b) Evaluate dy for the given values of x and dx. x = 0, dx = 0.05 dy Need Help? MY NOTES 27. [-/1 Points] DETAILS SCALCET9 3.10.033. Use a linear approximation (or differentials) to estimate the given number. (Round your answer to five decimal places.) √/28 ASK YOUR TEACHER PRACTICE ANOTHER

Answers

a) dy = (1/4) ex dx

b) the differential dy is 0.0125 when x = 0 and dx = 0.05.

To find the differential dy, given the function y=ex/2, we can use the following formula:

dy = (dy/dx) dx

We need to differentiate the given function with respect to x to find dy/dx.

Using the chain rule, we get:

dy/dx = (1/2) ex/2 * (d/dx) (ex/2)

dy/dx = (1/2) ex/2 * (1/2) ex/2 * (d/dx) (x)

dy/dx = (1/4) ex/2 * ex/2

dy/dx = (1/4) ex

Using the above formula, we get:

dy = (1/4) ex dx

Now, we can substitute the given values x = 0 and dx = 0.05 to find dy:

dy = (1/4) e0 * 0.05

dy = (1/4) * 0.05

dy = 0.0125

To learn more about function, refer:-

https://brainly.com/question/31062578

#SPJ11

Consider the parametric curve given by x = t³ - 12t, y=7t²_7 (a) Find dy/dx and d²y/dx² in terms of t. dy/dx = d²y/dx² = (b) Using "less than" and "greater than" notation, list the t-interval where the curve is concave upward. Use upper-case "INF" for positive infinity and upper-case "NINF" for negative infinity. If the curve is never concave upward, type an upper-case "N" in the answer field. t-interval:

Answers

(a) dy/dx:

To find dy/dx, we differentiate the given parametric equations x = t³ - 12t and y = 7t² - 7 with respect to t and apply the chain rule

(b) Concave upward t-interval:

To determine the t-interval where the curve is concave upward, we need to find the intervals where d²y/dx² is positive.

(a) To find dy/dx, we differentiate the parametric equations x = t³ - 12t and y = 7t² - 7 with respect to t. By applying the chain rule, we calculate dx/dt and dy/dt. Dividing dy/dt by dx/dt gives us the derivative dy/dx.

For d²y/dx², we differentiate dy/dx with respect to t. Differentiating the numerator and denominator separately and simplifying the expression yields d²y/dx².

(b) To determine the concave upward t-interval, we analyze the sign of d²y/dx². The numerator of d²y/dx² is -42t² - 168. As the denominator (3t² - 12)² is always positive, the sign of d²y/dx² solely depends on the numerator. Since the numerator is negative for all values of t, d²y/dx² is always negative. Therefore, the curve is never concave upward, and the t-interval is denoted as "N".

To learn more about curve  Click Here: brainly.com/question/32496411

#SPJ11

Evaluate the integral: S dz z√/121+z² If you are using tables to complete-write down the number of the rule and the rule in your work.

Answers

Evaluating the integral using power rule and substitution gives:

[tex](121 + z^{2}) ^{\frac{1}{2} } + C[/tex]

How to evaluate Integrals?

We want to evaluate the integral given as:

[tex]\int\limits {\frac{z}{\sqrt{121 + z^{2} } } } \, dz[/tex]

We can use a substitution.

Let's set u = 121 + z²

Thus:

du = 2z dz

Thus:

z*dz = ¹/₂du

Now, let's substitute these expressions into the integral:

[tex]\int\limits {\frac{z}{\sqrt{121 + z^{2} } } } \, dz = \int\limits {\frac{1}{2} } \, \frac{du}{\sqrt{u} }[/tex]

To simplify the expression further, we can rewrite as:

[tex]\int\limits {\frac{1}{2} } \, u^{-\frac{1}{2}} {du}[/tex]

Using the power rule for integration, we finally have:

[tex]u^{\frac{1}{2}} + C[/tex]

Plugging in 121 + z² for u gives:

[tex](121 + z^{2}) ^{\frac{1}{2} } + C[/tex]

Read more about Evaluating Integrals at: https://brainly.com/question/22008756

#SPJ4

70-2 Is λ=8 an eigenvalue of 47 7? If so, find one corresponding eigenvector. -32 4 Select the correct choice below and, if necessary, fill in the answer box within your choice. 70-2 Yes, λ=8 is an eigenvalue of 47 7 One corresponding eigenvector is A. -32 4 (Type a vector or list of vectors. Type an integer or simplified fraction for each matrix element.) 70-2 OB. No, λ=8 is not an eigenvalue of 47 7 -32 4

Answers

The correct answer is :Yes, λ=8 is an eigenvalue of 47 7 One corresponding eigenvector is A. -32 4 (Type a vector or list of vectors. Type an integer or simplified fraction for each matrix element.) The corresponding eigenvector is A= [ 7/8; 1].

Given matrix is:

47 7-32 4

The eigenvalue of the matrix can be found by solving the determinant of the matrix when [A- λI]x = 0 where λ is the eigenvalue.

λ=8 , Determinant = |47-8 7|

= |39 7||-32 4 -8|  |32 4|

λ=8 is an eigenvalue of the matrix [47 7; -32 4] and the corresponding eigenvector is:

A= [ 7/8; 1]

Therefore, the correct answer is :Yes, λ=8 is an eigenvalue of 47 7

One corresponding eigenvector is A. -32 4 (Type a vector or list of vectors. Type an integer or simplified fraction for each matrix element.)

The corresponding eigenvector is A= [ 7/8; 1].

To know more about eigenvector visit:

https://brainly.com/question/32593196

#SPJ11

A rumor spreads in a college dormitory according to the model dR R = 0.5R (1- - dt 120 where t is time in hours. Only 2 people knew the rumor to start with. Using the Improved Euler's method approximate how many people in the dormitory have heard the rumor after 3 hours using a step size of 1?

Answers

The number of people who have heard the rumor after 3 hours of using Improved Euler's method with a step size of 1 is R(3).  

The Improved Euler's method is a numerical approximation technique used to solve differential equations. It involves taking small steps and updating the solution at each step based on the slope at that point.

To approximate the number of people who have heard the rumor after 3 hours, we start with the initial condition R(0) = 2 (since only 2 people knew the rumor to start with) and use the Improved Euler's method with a step size of 1.

Let's perform the calculation step by step:

At t = 0, R(0) = 2 (given initial condition)

Using the Improved Euler's method:

k1 = 0.5 * R(0) * (1 - R(0)/120) = 0.5 * 2 * (1 - 2/120) = 0.0167

k2 = 0.5 * (R(0) + 1 * k1) * (1 - (R(0) + 1 * k1)/120) = 0.5 * (2 + 1 * 0.0167) * (1 - (2 + 1 * 0.0167)/120) = 0.0166

Approximate value of R(1) = R(0) + 1 * k2 = 2 + 1 * 0.0166 = 2.0166

Similarly, we can continue this process for t = 2, 3, and so on.

For t = 3, the approximate value of R(3) represents the number of people who have heard the rumor after 3 hours.

Learn more about Improved Euler's method here:

https://brainly.com/question/30860703

#SPJ11

For each series, state if it is arithmetic or geometric. Then state the common difference/common ratio For a), find S30 and for b), find S4 Keep all values in rational form where necessary. 2 a) + ²5 + 1² + 1/35+ b) -100-20-4- 15 15

Answers

a) The series is geometric. The common ratio can be found by dividing any term by the previous term. Here, the common ratio is 1/2 since each term is obtained by multiplying the previous term by 1/2.

b) The series is arithmetic. The common difference can be found by subtracting any term from the previous term. Here, the common difference is -20 since each term is obtained by subtracting 20 from the previous term.

To find the sum of the first 30 terms of series (a), we can use the formula for the sum of a geometric series:

Sₙ = a * (1 - rⁿ) / (1 - r)

Substituting the given values, we have:

S₃₀ = 2 * (1 - (1/2)³⁰) / (1 - (1/2))

Simplifying the expression, we get:

S₃₀ = 2 * (1 - (1/2)³⁰) / (1/2)

To find the sum of the first 4 terms of series (b), we can use the formula for the sum of an arithmetic series:

Sₙ = (n/2) * (2a + (n-1)d)

Substituting the given values, we have:

S₄ = (4/2) * (-100 + (-100 + (4-1)(-20)))

Simplifying the expression, we get:

S₄ = (2) * (-100 + (-100 + 3(-20)))

Please note that the exact values of S₃₀ and S₄ cannot be determined without the specific terms of the series.

Learn more about arithmetic series here: brainly.com/question/14203928

#SPJ11

The line AB passes through the points A(2, -1) and (6, k). The gradient of AB is 5. Work out the value of k.​

Answers

Answer:

Step-by-step explanation:

gradient = 5 = [k-(-1)]/[6-2]

[k+1]/4 = 5

k+1=20

k=19

Final answer:

The value of k in the line that passes through the points A(2, -1) and (6, k) with a gradient of 5 is found to be 19 by using the formula for gradient and solving the resulting equation for k.

Explanation:

To find the value of k in the line that passes through the points A(2, -1) and (6, k) with a gradient of 5, we'll use the formula for gradient, which is (y2 - y1) / (x2 - x1).

The given points can be substituted into the formula as follows: The gradient (m) is 5. The point A(2, -1) will be x1 and y1, and point B(6, k) will be x2 and y2. Now, we set up the formula as follows: 5 = (k - (-1)) / (6 - 2).

By simplifying, the equation becomes 5 = (k + 1) / 4. To find the value of k, we just need to solve this equation for k, which is done by multiplying both sides of the equation by 4 (to get rid of the denominator on the right side) and then subtracting 1 from both sides to isolate k. So, the equation becomes: k = 5 * 4 - 1. After carrying out the multiplication and subtraction, we find that k = 20 - 1 = 19.

Learn more about Line Gradient here:

https://brainly.com/question/30249498

#SPJ2

Fill the blanks to write general solution for a linear systems whose augmented matrices was reduce to -3 0 0 3 0 6 2 0 6 0 8 0 -1 <-5 0 -7 0 0 0 3 9 0 0 0 0 0 General solution: +e( 0 0 0 0 20 pts

Answers

The general solution is:+e(13 - e3 + e4  e5  -3e6 - 3e7  e8  e9)

we have a unique solution, and the general solution is given by:

x1 = 13 - e3 + e4x2 = e5x3 = -3e6 - 3e7x4 = e8x5 = e9

where e3, e4, e5, e6, e7, e8, and e9 are arbitrary parameters.

To fill the blanks and write the general solution for a linear system whose augmented matrices were reduced to

-3 0 0 3 0 6 2 0 6 0 8 0 -1 -5 0 -7 0 0 0 3 9 0 0 0 0 0,

we need to use the technique of the Gauss-Jordan elimination method. The general solution of the linear system is obtained by setting all the leading variables (variables in the pivot positions) to arbitrary parameters and expressing the non-leading variables in terms of these parameters.

The rank of the coefficient matrix is also calculated to determine the existence of the solution to the linear system.

In the given matrix, we have 5 leading variables, which are the pivots in the first, second, third, seventh, and ninth columns.

So we need 5 parameters, one for each leading variable, to write the general solution.

We get rid of the coefficients below and above the leading variables by performing elementary row operations on the augmented matrix and the result is given below.

-3 0 0 3 0 6 2 0 6 0 8 0 -1 -5 0 -7 0 0 0 3 9 0 0 0 0 0

Adding 2 times row 1 to row 3 and adding 5 times row 1 to row 2, we get

-3 0 0 3 0 6 2 0 0 0 3 0 -1 10 0 -7 0 0 0 3 9 0 0 0 0 0

Dividing row 1 by -3 and adding 7 times row 1 to row 4, we get

1 0 0 -1 0 -2 -2 0 0 0 -1 0 1 -10 0 7 0 0 0 -3 -9 0 0 0 0 0

Adding 2 times row 5 to row 6 and dividing row 5 by -3,

we get1 0 0 -1 0 -2 0 0 0 0 1 0 -1 10 0 7 0 0 0 -3 -9 0 0 0 0 0

Dividing row 3 by 3 and adding row 3 to row 2, we get

1 0 0 -1 0 0 0 0 0 0 1 0 -1 10 0 7 0 0 0 -3 -3 0 0 0 0 0

Adding 3 times row 3 to row 1,

we get

1 0 0 0 0 0 0 0 0 0 1 0 -1 13 0 7 0 0 0 -3 -3 0 0 0 0 0

So, we see that the rank of the coefficient matrix is 5, which is equal to the number of leading variables.

Thus, we have a unique solution, and the general solution is given by:

x1 = 13 - e3 + e4x2 = e5x3 = -3e6 - 3e7x4 = e8x5 = e9

where e3, e4, e5, e6, e7, e8, and e9 are arbitrary parameters.

Hence, the general solution is:+e(13 - e3 + e4  e5  -3e6 - 3e7  e8  e9)

The general solution is:+e(13 - e3 + e4  e5  -3e6 - 3e7  e8  e9)

learn more about coefficient matrix here

https://brainly.com/question/22964625

#SPJ11

Consider this function.

f(x) = |x – 4| + 6

If the domain is restricted to the portion of the graph with a positive slope, how are the domain and range of the function and its inverse related?

Answers

The domain of the inverse function will be y ≥ 6, and the range of the inverse function will be x > 4.

When the domain is restricted to the portion of the graph with a positive slope, it means that only the values of x that result in a positive slope will be considered.

In the given function, f(x) = |x – 4| + 6, the portion of the graph with a positive slope occurs when x > 4. Therefore, the domain of the function is x > 4.

The range of the function can be determined by analyzing the behavior of the absolute value function. Since the expression inside the absolute value is x - 4, the minimum value the absolute value can be is 0 when x = 4.

As x increases, the value of the absolute value function increases as well. Thus, the range of the function is y ≥ 6, because the lowest value the function can take is 6 when x = 4.

Now, let's consider the inverse function. The inverse of the function swaps the roles of x and y. Therefore, the domain and range of the inverse function will be the range and domain of the original function, respectively.

For more such questions on domain,click on

https://brainly.com/question/2264373

#SPJ8  

Other Questions
Suppose Y is the annual income, X is the number of years of education, and D is a dummy variable equaling 1 when a person is male. In a linear regression of Y on X and D, we estimate a coefficient of 3 for X and -4 for D. Then:a. The effect of being female is to lower income by 4.b. The effect of being male is to raise income by 4, holding education constant.c. The effect of being female is to raise income by 4, holding education constant.d. The effect of being female is to raise income by 4. After visiting several automobile dealerships, Richard selects the used car he wants. He likes its $15,300 price, but financing through the dealer is no bargain. He has $3,000 cash for a down payment, so he needs an $12,300 loan. In shopping at several banks for an installment loan, he learns that interest on most automobile loans is quoted at add-on rates. That is, during the life of the loan, interest is paid on the full amount borrowed even though a portion of the principal has been paid back. Richard borrows $12,300 for a period of four years at an add-on interest rate of 10 percent. a. What is the total interest on Richard's loan? Note: Do not round intermediate calculations. Round your answer to the nearest whole number. b. What is the total cost of the car? Note: Do not round intermediate calculations. Round your answer to the nearest whole number. c. What is the monthly poyment? Note: Do not round intermediate calculations. Round your answer to the nearest whole number. d. What is the annual percentage rate (APR)? Note: Do not round intermediate calculations. Enter your answer as a percent rounded to 2 decimal places. Consider the following. +1 f(x) = {x+ if x = -1 if x = -1 x-1 y 74 2 X -2 -1 2 Use the graph to find the limit below (if it exists). (If an answer does not exist, enter DNE.) lim, f(x) which partition management utility can be used to define and change various different guid example of a real-world problem you solved that required analysis.example of a real-world problem you solved that required decision making.example of a real-world problem you solved that required problem solving. Which of the following is a way that goals can be more effective?a) leaving golas in a more genatalized formb) only setting long term goalsc) examining goals closely to ensure that they have necessary componentsd) limiting goals to only one area of your life A Nacho Cheese company is considering hiring new VP of Marketing away from their main competitor. In the past, the company has had difficulty having the executives they've hired stick around longer than a year or two, and the performance of these recruits has only been moderately successful for the firm. As a result, the VP of HR wants to create a better contract that will help ensure this new hire seeks to maximize shareholder wealth rather than treating the company expense account like an ATM.Please name and describe three such provisions in the contract that would help meet that goal AND explain how they help to meet those goals.*Responses can be no less than 350 words. all of the following are true regarding bank statements except True or False/Explain: The Production Possibilities Frontier (PPF) Model and the Consumer Budget Constraint (CBC) Model explain the choices of firms and households. How do you record a sales tax payment in QuickBooks Online?Taxes > Sales Tax > Prepare return > View report > Enter payment amount, date and bank account > Record payment+ New > File Sales Tax > Enter payment amount, date and bank account > Record paymentTaxes > Sales Tax > View return > Record payment > Enter payment amount, date and bank account > Record payment+ New > File Sales Tax > Select state > Enter adjustment > Enter date and bank account > Record paymentTaxes > Sales Tax > View return > Select link for state website > Enter adjustments if needed > Enter payment date > Record payment Do this in two ways: (a) directly from the definition of the observability matrix, and (b) by duality, using Proposition 4.3. Proposition 5.2 Let A and T be nxn and C be pxn. If (C, A) is observable and T is nonsingular, then (T-AT, CT) is observable. That is, observability is invariant under linear coordinate transformations. Proof. The proof is left to Exercise 5.1. During 2020, Apple Corporation had the following share transactions:- Acquired 2,450 ordinary shares to be held as treasury shares for 110,250- Acquired additional 950 ordinary shares at 42.50 per share to be held as treasury shares- Sold 2,590 treasury shares at 40 per shareBy how much would total accumulated profits change because of the above transactions? Encode the answer as a negative if it's a decrease. A firm has the following capital structure. The corporate tax rate is 21%. Determine the after-tax weighted average cost of capital for the firm.Type Amount Return WeightMortgages (debt) 25,000,000 0.05 0.053Bonds 200,000,000 0.08 0.213Common Stock 175,000,000 0.1 0.372Preferred Stock 50,000,000 0.08 0.106Retained Earnings 120,000,000 0.12 0.255Group of answer choices1. 7.59%2. 9.19%3. 9.61%4. 5.17%Please show workings. thank you ZAG, a Utah company that makes protective gears for personal devices, has inventory of $193,000, equity of $395,100, total assets of $578,800, and sales of $612,300. What is the common-size percentage for the inventory account?the answer is not 33.34 which mozart opera was a huge success with viennese audiences? True/False The following pair of investment plans are identical except for a small difference in interest rates. Compute the balance in the accounts after 10 and 30 years. Discuss the difference. Chang invests $1300 in a savings account that earns 3.75% compounded annually. Kio invests $1300 in a different savings account that earns 4.0% compounded annually.After 10 years Chang will have a balance of approximately $After 30 years Chang will have a balance of approximately $(Round to the nearest cent as needed.)After 10 years Kio will have a balance of approximately $After 30 years Kio will have a balance of approximately $(Round to the nearest cent as needed.)After 10 years Kio will have $ or % More than Chang.After 30 years Kio will have $ or % More than Chang. for much of his music, aaron copland found inspiration in The denatid fer a product over six periods are \( 10,40,95,70,120 \), and 50 , respectively. In additiom, 1,1 and \( f, 7 \) are calculated 25120 and \( 247.5 \) ' 12 , respectively, where \( f \), is Calculate Income Tax using the methodology provided in Tax Calculation SampleIncome for the Year 2022 is $115,152 If the cost function is C(q)=1600+60q2What is the value of q at the minimum of Average Total Cost